Những câu hỏi liên quan
Thùy Hoàng
Xem chi tiết
Nguyễn Nhật Minh
5 tháng 8 2016 lúc 23:46

1. \(1=x^2+y^2\ge2xy\Rightarrow xy\le\frac{1}{2}\)

 \(A=-2+\frac{2}{1+xy}\ge-2+\frac{2}{1+\frac{1}{2}}=-\frac{2}{3}\)

max A = -2/3 khi x=y=\(\frac{\sqrt{2}}{2}\)

Bình luận (0)
Nguyễn Nhật Minh
5 tháng 8 2016 lúc 23:51

\(\frac{1}{xy}+\frac{1}{xz}=\frac{1}{x}\left(\frac{1}{y}+\frac{1}{z}\right)\ge\frac{1}{x}.\frac{4}{y+z}=\frac{4}{\left(4-t\right)t}=\frac{4}{4-\left(t-2\right)^2}\ge1\) với t = y+z => x =4 -t

Bình luận (0)
Nguyễn Nhật Minh
6 tháng 8 2016 lúc 0:08

\(A=x^2+y^2=\frac{\left(1^2+1^2\right)\left(x^2+y^2\right)}{2}\ge\frac{\left(1.x+1.y\right)^2}{2}=\frac{1}{2}\)A min = 1 khi x =y = 1/2

\(\sqrt{A}=\sqrt{x^2+y^2}\le\sqrt{x^2}+\sqrt{y^2}=x+y=1\)\(\sqrt{a+b}\le\sqrt{a}+\sqrt{b}\))

=> A\(\le1\) => Max A = 1 khi x =0;y =1 hoặc x =1 ; y =0

Bình luận (0)
Diệp Nguyễn Thị Huyền
Xem chi tiết
Đặng Ngọc Quỳnh
19 tháng 7 2021 lúc 19:06

Theo đề bài, ta có:

\(x^3+y^3=x^2-xy+y^2\)

hay \(\left(x^2-xy+y^2\right)\left(x+y-1\right)=0\)

\(\Rightarrow\orbr{\begin{cases}x^2-xy+y^2=0\\x+y=1\end{cases}}\)

+ Với \(x^2-xy+y^2=0\Rightarrow x=y=0\Rightarrow P=\frac{5}{2}\)

+ với \(x+y=1\Rightarrow0\le x,y\le1\Rightarrow P\le\frac{1+\sqrt{1}}{2+\sqrt{0}}+\frac{2+\sqrt{1}}{1+\sqrt{0}}=4\)

Dấu đẳng thức xảy ra <=> x=1;y=0 và \(P\ge\frac{1+\sqrt{0}}{2+\sqrt{1}}+\frac{2+\sqrt{0}}{1+\sqrt{1}}=\frac{4}{3}\)

Dấu đẳng thức xảy ra <=> x=0;y=1

Vậy max P=4 và min P =4/3

Bình luận (0)
 Khách vãng lai đã xóa
Binh Hang
Xem chi tiết
Nguyễn Nhật Minh
Xem chi tiết
phan thị oanh
15 tháng 12 2015 lúc 12:29

GTLN =3

GTNN = 1

Bình luận (0)
Phác Đại Nhân
Xem chi tiết
Dũng Lương Trí
Xem chi tiết
Xem chi tiết
Hồ Thị Mai Linh
Xem chi tiết
Thu Nguyễn
Xem chi tiết
tth_new
12 tháng 12 2018 lúc 18:01

\(A=\frac{x}{x+1}+\frac{y}{y+1}+\frac{z}{z+1}\).Áp dụng BĐT Cauchy-Schwarz,ta có:

\(=\left(1-\frac{1}{x+1}\right)+\left(1-\frac{1}{y+1}\right)+\left(1-\frac{1}{z+1}\right)\)

\(=\left(1+1+1\right)-\left(\frac{1}{x+1}+\frac{1}{y+1}+\frac{1}{z+1}\right)\)

\(\ge3-\frac{9}{\left(x+y+z\right)+\left(1+1+1\right)}=\frac{3}{4}\)

Dấu "=" xảy ra khi x = y = z = 1/3

Vậy A min = 3/4 khi x=y=z=1/3

Bình luận (0)
tth_new
12 tháng 12 2018 lúc 18:01

Bỏ chữ "Áp dụng bđt Cauchy-Schwarz,ta có:"giùm mình,nãy đánh nhầm ở bài làm trước mà quên xóa đi!

Bình luận (0)
tth_new
12 tháng 12 2018 lúc 18:04

À mà để phải là tìm Max mới đúng chứ nhỉ?

Do đó,bạn sửa dòng: \(\ge3-\frac{9}{\left(x+y+z\right)+\left(1+1+1\right)}=\frac{3}{4}\) đến hết thành:

"\(\le3-\frac{9}{\left(x+y+z\right)+\left(1+1+1\right)}=\frac{3}{4}\)

Dấu "=" xảy ra khi x=y=z=1/3

Vậy A max = 3/4 khi x=y=z=1/3

Bình luận (0)